Difference between revisions of "2011 AMC 12A Problems/Problem 20"

(Created page with '== Problem == == Solution == == See also == {{AMC12 box|year=2011|num-b=19|num-a=21|ab=A}}')
 
(Problem)
Line 1: Line 1:
 
== Problem ==
 
== Problem ==
 +
Let <math>f(x)=ax^2+bx+c</math>, where <math>a</math>, <math>b</math>, and <math>c</math> are integers. Suppose that <math>f(1)=0</math>, <math>50<f(7)<60</math>, <math>70<f(8)<80</math>, <math>5000k<f(100)<5000(k+1)</math> for some integer <math>k</math>. What is <math>k</math>?
 +
 +
<math>
 +
\textbf{(A)}\ 1 \qquad
 +
\textbf{(B)}\ 2 \qquad
 +
\textbf{(C)}\ 3 \qquad
 +
\textbf{(D)}\ 4 \qquad
 +
\textbf{(E)}\ 5 </math>
 +
 
== Solution ==
 
== Solution ==
 
== See also ==
 
== See also ==
 
{{AMC12 box|year=2011|num-b=19|num-a=21|ab=A}}
 
{{AMC12 box|year=2011|num-b=19|num-a=21|ab=A}}

Revision as of 02:36, 10 February 2011

Problem

Let $f(x)=ax^2+bx+c$, where $a$, $b$, and $c$ are integers. Suppose that $f(1)=0$, $50<f(7)<60$, $70<f(8)<80$, $5000k<f(100)<5000(k+1)$ for some integer $k$. What is $k$?

$\textbf{(A)}\ 1 \qquad \textbf{(B)}\ 2 \qquad \textbf{(C)}\ 3 \qquad \textbf{(D)}\ 4 \qquad \textbf{(E)}\ 5$

Solution

See also

2011 AMC 12A (ProblemsAnswer KeyResources)
Preceded by
Problem 19
Followed by
Problem 21
1 2 3 4 5 6 7 8 9 10 11 12 13 14 15 16 17 18 19 20 21 22 23 24 25
All AMC 12 Problems and Solutions